K
Khách

Hãy nhập câu hỏi của bạn vào đây, nếu là tài khoản VIP, bạn sẽ được ưu tiên trả lời.

7 tháng 6 2019

Ta có: \(\frac{1}{5}\)>\(\frac{1}{14}\)

\(\frac{1}{14}\)=\(\frac{1}{14}\)

\(\frac{1}{28}\)<\(\frac{1}{14}\)

...

\(\frac{1}{97}< \frac{1}{14}\)

=>Cả dãy số < \(\frac{1}{14}.7\)<\(\frac{1}{2}\)

15 tháng 7 2019

Hai dãy số trên chưa hai bđt ngược chiều: 1/5 > 1/14 và 1/28<1/14 ..... thì làm sao mà cộng theo vế để suy ra như vậy đc?

15 tháng 7 2019

Hỏi đáp Toán

25 tháng 3 2017

Đặt \(\frac{1}{5}+\frac{1}{14}+\frac{1}{28}+\frac{1}{44}+\frac{1}{61}+\frac{1}{85}+\frac{1}{97}=A\)

Ta có : \(A=\frac{1}{5}+\left(\frac{1}{14}+\frac{1}{28}+\frac{1}{44}\right)+\left(\frac{1}{61}+\frac{1}{85}+\frac{1}{97}\right)\)

\(A< \frac{1}{5}\left(\frac{1}{14.3}\right)+\left(\frac{1}{61.3}\right)\)

\(A< \frac{1}{5}+\frac{3}{14}+\frac{3}{61}\)

\(A< \frac{1}{5}+\frac{3}{12}+\frac{1}{20}\)

\(A< \frac{1}{2}\left(ĐPCM\right).\)

19 tháng 9 2016

Ta thấy các phân số trên khi quy đồng mẫu số chứa lũy thừa của 2 với số mũ cao nhất là 24

Như vậy, các phân số trên khi quy đồng mẫu số sẽ có tử chẵn, chỉ có phân số 1/16 có tử lẻ

=> tổng trên có tử lẻ, mẫu chẵn, không là số nguyên (đpcm)

C` cách 2 nhưng dài hơn

26 tháng 3 2017

\(\frac{1}{5}+\frac{1}{14}+\frac{1}{28}+\frac{1}{44}+\frac{1}{61}+\frac{1}{85}+\frac{1}{97}< \frac{1}{2}\)

\(\frac{1}{5}+\frac{1}{14}+\frac{1}{28}+\frac{1}{44}+\frac{1}{61}+\frac{1}{85}+\frac{1}{97}=0,36833......\)

mà \(\frac{1}{2}=0,5\)

\(0,36833..< 0,5\)

Vậy \(\frac{1}{5}+\frac{1}{14}+\frac{1}{28}+\frac{1}{44}+\frac{1}{61}+\frac{1}{85}+\frac{1}{97}< \frac{1}{2}\)

26 tháng 3 2017

Đặt \(\frac{1}{5}+\frac{1}{14}+\frac{1}{28}+\frac{1}{44}+\frac{1}{61}+\frac{1}{85}+\frac{1}{97}=A\)

Ta có : \(A=\frac{1}{5}+\left(\frac{1}{14}+\frac{1}{28}+\frac{1}{44}\right)+\left(\frac{1}{61}+\frac{1}{85}+\frac{1}{97}\right)\)

\(A< \frac{1}{5}\left(\frac{1}{14.3}\right)+\left(\frac{1}{61.3}\right)\)

\(A< \frac{1}{5}+\frac{3}{14}+\frac{3}{61}\)

\(A< \frac{1}{5}+\frac{3}{12}+\frac{1}{20}\)

\(A< \frac{1}{2}\left(ĐPM\right)\).